Managerial Economics Final Exam

¡Supera tus tareas y exámenes ahora con Quizwiz!

Suppose the growth rate of the firm's profit is 7 percent, the interest rate is 10 percent, and the current profits of the firm are $120 million. What is the value of the firm? Current profits *{(1+interest rate)/(interest rate-growth rate)

$ 4,400 million

The price elasticity of demand for senior citizens purchasing coffee from McDonald's is −5, while non-senior citizens have a price elasticity of demand equal to −1.25. If it costs McDonald's $0.02 to produce a coffee, the optimal price for a cup of coffee for senior citizens and the resultant marginal cost under third degree price discrimination are, respectively $0.016 and $0.20. $0.02 and $0.80. $0.025 and $0.02. $0.10 and $0.02

$0.025 and $0.02.

A local video store estimates its average customer's demand per year is Q = 7 − 2P, and it knows the marginal cost of each rental is $0.5. How much should the store charge for each rental if it engages in optimal two-part pricing? $0.35 $0.5 $0.7 $1.00

$0.5

Suppose the cost function is C(Q) = 50 + Q − 10Q2 + 2Q3. What is the total cost of producing 10 units? $2,060 $1,060 $560 $1,010

$1,060

If the annual interest rate is 0 percent, the present value of receiving $1.10 in the next year is:

$1.10

You are the manager of a firm that produces output in two plants. The demand for your firm's product is P = 20 − Q, where Q = Q1 + Q2. The marginal costs associated with producing in the two plants are MC1 = 2 and MC2 = 2Q2. What is the profit-maximizing price that the firm should charge? $8 $9 $11 $12

$11

You are a manager in a perfectly competitive market. The price in your market is $14. Your total cost curve is C(Q) = 10 + 4Q + 0.5Q2. What price should you charge in the short run? $12 $14 $16 $18

$14

How much would the monopoly in the figure below spend to prevent its price being regulated to marginal (or average) cost? $16 $8 $4 $0

$16

A firm with market power has an individual consumer demand of Q = 20 − 4P and costs of C = 4Q. What is optimal price to charge for a block of 20 units? $18 $36 $72 $90

$18

A monopoly produces widgets at a marginal cost of $8 per unit and zero fixed costs. It faces an inverse demand function given by P = 38 - Q. What are the profits of the monopoly in equilibrium? $225 $120 $345 None of the statements is correct.

$225

Consider the monopoly in the figure below with price regulated at $20 per unit. Consumer surplus at the regulated price is: $500. $3,920. $2,300. There is insufficient information to determine consumer surplus.

$3,920.

You are the manager of a Mom and Pop store that can buy milk from a supplier at $2.00 per gallon. If you believe the elasticity of demand for milk by customers at your store is −3, then your profit-maximizing price is: $1.33. $2.75. $3.00. $4.50.

$3.00.

The domestic demand and supply for sugar are Qd = 60,000 − 400P and QSD = 20,000 + 500P. The foreign supply is QSF = 20,000 + 100P. Suppose an import quota of 13,000 is imposed in the domestic market. What will be the new market price of sugar? $15 $20 $30 $45

$30

A local video store estimates its average customer's demand per year is Q = 20 − 4P, and it knows the marginal cost of each rental is $1.00. How much should the store charge for an annual membership in order to extract the entire consumer surplus via an optimal two-part pricing strategy? $20 $32 $40 $64

$32

Two identical firms compete as a Cournot duopoly. The inverse market demand they face is P = 80 − 4Q. The cost function for each firm is C(Q) = 8Q. The price charged in this market will be $12. $32. $48. $56.

$32.

You are a hotel manager and you are considering four projects that yield different payoffs, depending upon whether there is an economic boom or a recession. The potential payoffs and corresponding payoffs are summarized in the following table. The expected value of project B is $5. $10. $20. None of the answers are correct.

$5

Consider a market characterized by the following inverse demand and supply functions and supply functions: PX=40-4QX and PX = 10+2QX. Compute the surplus received by consumers and producers -$25 and $25, respectively -$20 and $40, respectively -$40 and $20, respectively -$50 and $25, respectively

$50 and $25, respectively 40-4QX=10+2QX. =30=6QX QX=5 (quantity) PX= 40-4QX = 40-4(5) PX =20 (equilibrium price) PX= 40-4QX. PX= 10+2QX =40-4(0). =10+2(0) PX=40. PX=10 1/2*(40-20)*5 1/2*(20-10)*5 1/2*20*5. 1/2*10*5 Consumer surplus = 50. =25

According to this section's readings, what was the cost of the 2001 Enron bankruptcy? $66 Billion $70 Billion $74 Billion $78 Billion

$74 Billion

In the game shown below, firms 1 and 2 must independently decide whether to charge high or low prices Which of the following are Nash equilibrium payoffs in the one-shot game? (0, 0) (5, -5) (-5, 5) (10, 10)

(10, 10)

In the game shown below, firms 1 and 2 must independently decide whether to charge high or low prices. Suppose the game is infinitely repeated. Then the "best" the firms could do in a Nash equilibrium is to earn _________ per period (0, 0) (5, -5) (-5, 5) (10, 10)

(10, 10)

Refer to the following game. What are the Nash equilibrium strategies for firm A and firm B, respectively, in a one-shot game? (low price, low price) (high price, high price) (low price, high price) (low price, low price) and (high price, high price)

(low price, low price)

Suppose you read in an industry publication that the Rothschild index for the petroleum industry is 0.88. Based on past experience, you know that the price elasticity of demand for the petroleum products sold by your firm is −1.5. Based on this information, you know that the elasticity of demand for a representative firm in the petroleum industry is: 1.32. 1.70. −0.587. −1.32.

-1.32

The demand function in the accompanying table is QXd = 100-2PX. Based on this information, when QX = 80, the price, PX (point A) is: -5 -10 -15 -20

10

The demand for good X has been estimated to be in Qxd = 100 - 2.5 in PX + 4 in PY + in M. The own price elasticity of good X is: - (-2.5) -4.0 -(-2.5 percent) -4.0 percent

-2.5

which of the following is a correct statement about the own price elasticity of demand -demand tends to be more inelastic in the short term than the long term -demand tends to be more elastic as more substitutes are available -demand tends to be more inelastic for goods that comprise a smaller share of a consumers budget -all of the statements are correct

-all of the statements are correct

A firm derives revenue from two sources: goods X and Y. Annual revenues from good X and Y are $10,000 and $20,000, respectively. If the price elasticity of demand for good X is -4.0 and the cross-price elasticity of demand between Y and X is 2.0, then a 2 percent decrease in the price of X will: -increase total revenues from X and Y by $520 -decrease total revenues from X and Y by $200 -leave total revenues from X and Y unchanged -decrease total revenues for X and Y by $600

-decrease total revenues from X and Y by $200

Suppose two types of consumers buy suits. Consumers of type A will pay $100 for a coat and $50 for pants. Consumers of type B will pay $75 for a coat and $75 for pants. The firm selling suits faces no competition and has a marginal cost of zero. If the firm sells coats and pants for $25 each, but offers a bundle containing both a coat and pants for $150, how many bundles will the firm sell? 0 1 2 Insufficient information

0

You are a hotel manager and you are considering four projects that yield different payoffs, depending upon whether there is an economic boom or a recession. The potential payoffs and corresponding payoffs are summarized in the following table. If a manager adopted both project A and B simultaneously, the variance in returns associated with this joint project would be 0. 10. 30. 500

0

Suppose each of the 50 states had only one gasoline station, and all stations were the same size. The four-firm concentration ratio for the state of New York, based on the state data, is: 1.0. 0.08. 0.32. 0.16.

0.08.

Suppose the demand function is Qxd = 100-8Px+6Py-M if Px = $4, Py=$2 and M=$10, what is the cross-price elasticity of good x with respect to the price of good y? -0.17 -0.38 -0.21 -0.04

0.17

An industry consists of six firms with annual sales of $300, $500, $400, $700, $600, and $600. What is the industry's C4? 0.58 0.62 0.74 0.77

0.77

A fair coin is flipped. You will be paid $1 when it is heads and penalized $1 otherwise. What is the variance of the payoffs? 0. 1. 0.50. 0.25.

1

You are the manager of a firm that produces output in two plants. The demand for your firm's product is P = 78 − 15Q, where Q = Q1 + Q2. The marginal costs associated with producing in the two plants are MC1 = 3Q1 and MC2 = 2Q2. How much output should be produced in plant 1 in order to maximize profits? 1 2 3 4

1

Suppose that there are two industries, A and B. There are five firms in industry A with sales at $5 million, $2 million, $1 million, $1 million, and $1 million, respectively. There are four firms in industry B with equal sales of $2.5 million for each firm. The four-firm concentration ratio for industry B is: 0.9. 1.0. 0.8. 0.7

1.0.

Consider the monopoly in the figure below with price regulated at $20 per unit. In this market, ___________ units will be exchanged. 10 25 42 50

10

The external marginal cost of producing coal is MCexternal = 6Q while the internal marginal cost is MCinternal = 4Q. The inverse demand for coal is given by P = 120 − 2Q. What is the socially efficient level of output? 10 20 15 8

10

You are a hotel manager and you are considering four projects that yield different payoffs, depending upon whether there is an economic boom or a recession. The potential payoffs and corresponding payoffs are summarized in the following table. If a manager adopted both project A and B simultaneously, the expected value of this joint project would be: 10. 20. 30. 40.

10

What is the level of net benefits when 20 units are produced?

100

You are the manager of a monopoly that faces a demand curve described by P = 230 − 20Q. Your costs are C = 5 + 30Q. The profit-maximizing price is: 150. 90. 130. 110.

130

What is the net benefit associated with producing two units of the control variable, Q (identify point C in the table)

1400

If the income elasticity for lobster is 0.4, a 40 percent increase in income will lead to a: -10 percent drop in demand for lobster -16 percent increase in demand for lobster -20 percent increase in demand for lobster -4percent increase in demand for lobster

16 percent increase in demand for lobster (.4*.4=16%)

You are the manager of a monopoly that faces a demand curve described by P = 63 − 5Q. Your costs are C = 10 + 3Q. Your firm's maximum profits are: 0. 66. 120. 170

170

Consider two firms competing to sell a homogeneous product by setting price. The inverse demand curve is given by P = 6 − Q. If each firm's cost function is Ci (Qi ) = 6 + 2Qi , then each firm will symmetrically produce _________ units of output and earn ___________ 4 units; profits of $6 2 units; profits of $2 4 units; losses of $2 2 units; losses of $6

2 units; losses of $6

You are the manager of a popular hat company. You know that advertising elasticity of demand for your product is 0.25. How much will you have to increase advertising in order to increase demand by 5 percent -0.05 percent -20 percent -25 percent -1.25 percent

20 percent

Refer to the normal-form game of price competition in the payoff matrix below Suppose the game is infinitely repeated, and the interest rate is 10 percent. Both firms agree to charge a high price, provided no player has charged a low price in the past. If both firms stick to this agreement, then the present value of firm A's payoffs are: 220. 110. 330. 550.

220.

You are a hotel manager and you are considering four projects that yield different payoffs, depending upon whether there is an economic boom or a recession. The potential payoffs and corresponding payoffs are summarized in the following table. The variance in the returns of project A is: 900. 225. 0. 1,600.

225

According to the table below, what is the total cost of producing 125 units of output? 1,000 2,050 1,400 2,400

2400

What is the total benefit associated with producing four units of the control variable, Q (identify point A in the table)

3000

What is the marginal cost of producing the tenth unit?

4

A risk-neutral monopoly must set output before it knows the market price. There is a 50 percent chance the firm's demand curve will be P = 40 − Q and a 50 percent chance it will be P = 60 − Q. The marginal cost of the firm is MC = 3Q. The expected profit-maximizing price is: $10. $20. $30. $40.

40

You are the manager of a firm that sells its product in a competitive market at a price of $40. Your firm's cost function is C = 60 + 4Q2. Your firm's maximum profits are 36. 60. 40. 80.

40

Suppose the market supply for good X is given by QXS-100+5PX. If the equilibrium price of X is $100 per unit then producers revenue from X is -100 -20,000 -40,000 -cannot be determined from the information contained in the question

40,000

If a firm manager has a base salary of $100,000 and also receives 5 percent of all profits, what percentage of his/her final income will be from a profit-sharing plan when profit equals $1,500,000? 51 percent 27 percent 43 percent 48 percent

43 percent

Refer to the normal-form game of price competition in the payoff matrix below. Suppose that firm A deviates from a trigger strategy to support a high price. What is the present value of A's payoff from cheating 70 50 30 20

50

The demad for good X is estimated to be Qxd = 10,000 - 4PX + 5PY + AX where PX is the price of X, PY is the price of good Y, M is income, and AX is the amount of advertising on X. Suppose the present price of good X is $50, PY=$100, M = $25,000, and AX = 1000 units. What is the demand curve good X? -61500 -61300 -61300-4PX -61500-4PX

61500-4PX

Suppose the production function is given by Q = 4K + 6L. What is the average product of capital when 10 units of capital and 5 units of labor are employed? 14 10 7 5

7

According to the table below, what is the marginal cost of producing 90 units of output? 5.32 8.75 11.67 21.00

8.75

Suppose that the demand in a particular industry is given by Qd = 100 - 2P. When the market price in the industry is $10 per unit, total demand in the industry is ___. Furthermore, assume that each of the four largest firms in the industry sell 15 units. Based on this information, the four-firm concentration ratio is ___ 80 units;1.00 45 units;0.75 80 units; 0.75 45 units; 0.25

80 units; 0.75

The demand function in the accompanying table is QX d = 100 − 2PX. Based on this information, compute the total revenue when QX = 20 (point D) $750 $800 $850 $900

800

You are the manager of a firm that sells its product in a competitive market at a price of $50. Your firm's cost function is C = 40 + 5Q2. Your firm's maximum profits are: 125. 250. 100. 85.

85

Which of the following is true? A Nash equilibrium is always perfect. A perfect equilibrium is always Nash. A Nash equilibrium is always perfect in a multistage game. Perfect equilibrium and Nash equilibrium are the same concept but with different name

A perfect equilibrium is always Nash.

Suppose option A has a higher expected value than option B. Which of the following statements is, in general, true? A risk-averse person prefers option B to option A. A risk-neutral person is indifferent between options A and B. A risk-averse person prefers option A to option B. Insufficient information to determine.

A risk-averse person prefers option B to option A

Which of the following is used to measure market structure and performance? Four-firm concentration ratio HHI (Herfindahl-Hirschman index) Dansby-Willig Performance index All of the answers are correct.

All of the answers are correct

To engage in first-degree price discrimination, a firm mus be able to set P > MC. know each consumer's maximum willingness to pay. prevent low-value consumers from reselling to high-value consumers. All of the answers are correct

All of the answers are correct.

One of the conditions under which price discrimination is profitable is: ability to identify consumer types. inability to resell the good. differences in demand elasticities. All of the statements associated with this question are correct

All of the statements associated with this question are correct.

Shirking can take the form of: Long lunch hours Sleeping at work Leaving work early All of the statements associated with this question are correct

All of the statements associated with this question are correct.

Under the merger guidelines written by the DOJ and FTC, a merger may not be challenged if: there is significant foreign competition. the firms involved have monetary problems. there is an emergence of new technology. All of the statements associated with this question are correct.

All of the statements associated with this question are correct.

Which type of compensation method works by performance bonus? Profit sharing Revenue sharing Piece rate All of the statements associated with this question are correct.

All of the statements associated with this question are correct.

Refer to the normal-form game of bargaining shown below. Suppose that management and the union are bargaining over how much of a $500 surplus to give to the union. It is assumed that the surplus can only be split into $250 increments. Furthermore, negotiations are set up such that management and the union must simultaneously and independently write down the amount of surplus to allocate to the union. The payoff structure to this oneshot bargaining game is listed in Figure 10-16. Find the Nash equilibrium(ia) to this game Union write down $0 and management write down $500. Union write down $250 and management write down $250. Union write down $500 and management write down $0. All of the statements associated with this question constitute Nash equilibria.

All of the statements associated with this question constitute Nash equilibria.

Refer to the normal-form game of price competition shown below. For what values of x is strategy (B, D) the only Nash equilibrium of the game? All x > 450 All x < 450 x = 450 x < 50

All x > 450

Refer to the normal-form game of price competition shown below. For what values of x is strategy D strictly dominant for firm B? All x > 450 All x < 450 x = 450 x < 50

All x > 450

In perfect competition, which is NOT true? Both concentration ratios and Rothschild indexes tend to be close to zero. There are a large number of firms, and each is small relative to the entire market. At least one firm has a perceptible impact on the market price. Firms produce homogenous goods

At least one firm has a perceptible impact on the market price

Which of the following is NOT a measure of productivity? Total product Marginal product Average advertising Input-output ratio

Average advertising

A duopoly in which both firms have a Lerner index of monopoly power equal to 0 is probably a Sweezy oligopoly. Cournot oligopoly. Stackelberg oligopoly. Bertrand oligopoly.

Bertrand oligopoly

Which of the following is a means of eliminating the undesirable effects of adverse selection A long-term relationship Writing a contract to guarantee the quality Both a long-term relationship and writing a contract to guarantee the quality None of the statements is correct

Both a long-term relationship and writing a contract to guarantee the quality

An ad valorem tax shifts the supply curve -down by the amount of the tax -up by the amount of the tax -by rotating it counter-clockwise -by rotating it clockwise

By rotating it counter-clockwise

You are a hotel manager and you are considering four projects that yield different payoffs, depending upon whether there is an economic boom or a recession. The potential payoffs and corresponding payoffs are summarized in the following table. A B C D

C

The production function Q = L.5K .5 is called: Cobb Douglas. Leontief. linear. None of the answers are correct.

Cobb Douglas

Which of the following is a strategy(ies) used by firms in monopolistically competitive industries to convince consumers that their product is better than their rivals' products? Comparative advertising Niche marketing Equity marketing Comparative advertising or niche marketing

Comparative advertising or niche marketing

You are a hotel manager and you are considering four projects that yield different payoffs, depending upon whether there is an economic boom or a recession. The potential payoffs and corresponding payoffs are summarized in the following table A. B. C. D

D

The import tariffs that President Bush placed on imported steel likely had what effect? Domestic steel producers were helped, but domestic steel consumers were hurt. Both domestic steel consumers and foreign steel producers were helped. Both domestic steel producers and domestic steel consumers were helped. Foreign steel producers were helped, but domestic steel consumers were hurt

Domestic steel producers were helped, but domestic steel consumers were hurt

What is implied when the total cost of producing Q1 and Q2 together is less than the total cost of producing Q1 and Q2 separately? Economies of scale Diminishing average fixed costs Cost complementarity Economies of scope

Economies of scope

Suppose P = 20 − 2Q is the market demand function for a local monopoly. The marginal cost is 2Q. The firm currently uses a standard pricing strategy. Which of the following will allow the firm to enhance the profits? Engage in two-part pricing. Engage in commodity bundling. Engage in randomized pricing. Engage in two-part pricing and engage in commodity bundling.

Engage in two-part pricing.

Your firm is planning to hold an auction to sell its oil field. What type of auction should you suggest to your boss Dutch auction English auction Second-price, sealed-bid auction None of the statements is correct

English auction

A new firm enters a market which is initially serviced by a Bertrand duopoly charging a price of $30. Assuming that the new firm is equally as efficient as the incumbent firms, what will the new price be should the three firms coexist after the entry? Above $30 Below $30 Equal to $30 Unable to tell given the information provided.

Equal to $30

Which of the following is true? In a one-shot game, a collusive strategy always represents a Nash equilibrium. A perfect equilibrium occurs when each player is doing the best he can regardless of what the other player is doing. Each Nash equilibrium is a perfect equilibrium. Every perfect equilibrium is a Nash equilibrium

Every perfect equilibrium is a Nash equilibrium.

Refer to the normal-form game of price competition shown below. Firm A must decide whether or not to introduce a new product. If firm A introduces a new product, firm B must decide whether or not to clone the product. The payoff structure of the game is depicted in Figure 10-12. The subgame perfect Nash equilibrium to this game is Firm A plays "Introduce"; firm B plays "Clone" if firm A plays "Introduce." Firm A plays "Do Not Introduce"; firm B plays "Clone" if firm A plays "Introduce." Firm A plays "Introduce"; firm B plays "Do Not Clone" if firm A plays "Introduce." Firm A plays "Do Not Introduce"; firm B plays "Do Not Clone" if firm A plays "Introduce."

Firm A plays "Do Not Introduce"; firm B plays "Clone" if firm A plays "Introduce."

Which of the following is true about an excise tariff? Domestic firms' marginal cost curves are shifted up by the amount of the excise tariff. Foreign firms' average cost curves are shifted down by the amount of the excise tariff. Domestic firms' average cost curves are shifted up by the amount of the excise tariff. Foreign firms' marginal cost curves are shifted up by the amount of the excise tariff.

Foreign firms' marginal cost curves are shifted up by the amount of the excise tariff.

Which of the following is true about an excise tariff? Domestic firms' marginal cost curves are shifted up by the amount of the excise tariff. Foreign firms' average cost curves are shifted down by the amount of the excise tariff. Domestic firms' average cost curves are shifted up by the amount of the excise tariff. Foreign firms' marginal cost curves are shifted up by the amount of the excise tariff

Foreign firms' marginal cost curves are shifted up by the amount of the excise tariff.

What is meant by free trade?

Free trade is a policy by which a government does not discriminate against imports or interfere with exports by applying tariffs or subsidies.

Which of the following is NOT a benefit associated with producing inputs within a firm? Reduction in transaction costs. Gains of specializing. Reductions in opportunism. Mitigation of hold-up problem

Gains of specializing

Which of the following is a correct statement about a Nash equilibrium in a two player game? The joint payoffs of the two players are highest compared to other strategy pairs. A Nash equilibrium is always unique in real-world problems. Given another player's strategy, no player can improve her welfare by unilaterally changing her strategy. All of the statements associated with this question are correct

Given another player's strategy, no player can improve her welfare by unilaterally changing her strategy.

In the video "Free Trade vs Protectionsim," (section Readings) what are the two examples given by Professor Boudreaux of free trade in action? China and India The United States of America and the United Kingdom Hong Kong and inter-state trade among the United States of America Germany and Japan

Hong Kong and inter-state trade among the United States of America

Which of the following integration types has the potential problem of increasing the firm's market power? Vertical integration Horizontal integration Cointegration Conglomerate integration

Horizontal integration

Which of the following is true? -In Bertrand oligopoly markets, each firm believes that its rivals will hold their output constant if it changes its output. -In Cournot oligopoly markets, firms produce an identical product at a constant marginal cost and engage in price competition. -In Sweezy oligopoly markets, each firm believes rivals will cut their prices in response to a price reduction, but will not raise prices in response to price increases. -In oligopoly markets, a change in marginal cost never has an effect on output or price.

In Sweezy oligopoly markets, each firm believes rivals will cut their prices in response to a price reduction, but will not raise prices in response to price increases.

In 2011 when the exchange rate between drams and rupees was 1.48 rupees/dram, the price of a roti was 120 rupees. In the next 8 years the price of a roti (in rupees) went up by 15%. The current exchange rate is 1.32 rupees/dram. By how much has the price of a roti increased/decreased in drams? Increase by 29% Decrease by 29% Increase by 2.6% Decrease by 2.6%

Increase by 29%

Which of the following is true about where a profit-maximizing monopoly will produce on a linear demand curve when it has positive marginal costs? It will produce output on the inelastic portion of the demand curve. It will produce output where MR < 0. It will produce output where MR = 0. It will produce output on the elastic portion of the demand curve.

It will produce output on the elastic portion of the demand curve

Suppose there is a simultaneous increase in demand and decrease in supply, what effect will this have on the equilibrium price -It will rise -it will fall -it may rise or fall -it will remain the same

It will rise

If insurance companies are required to offer coverage to all interested people, it is said that premiums for each person will be increased. Assume that the insurance market is perfectly competitive. What is the major reason for raising the premium? The insurance companies take advantage of the increased demand and collude. Medical services are more expensive because of increased demand. Less healthy people join the pool of insured and hence increase the risk and the premium. None of the statements is correct.

Less healthy people join the pool of insured and hence increase the risk and the premium.

MCI announced a price discount plan for small firms. Their stock immediately fell in price. This shows that MCI is probably competing in a Bertrand oligopolistic industry. stockholders are sometimes not rational. there is increased demand for MCI's stock. AT&T sold out its stock of MCI just after the announcement

MCI is probably competing in a Bertrand oligopolistic industry

Which of the following is a profit-maximizing condition for a Cournot oligopolist? MR = MC. Q1 = Q2 =... = Qn. P = MR. All of the statements associated with this question are correct.

MR = MC

Consider a Stackelberg duopoly with the following inverse demand function: P = 100 − 2Q1 − 2Q2. The firms' marginal costs are identical and are given by MCi = 2. Based on this information, the Stackelberg follower's marginal revenue function is: MRF(QL,QF) = 100 − 2QL − 4QF. MRF(QL,QF) = 100 − 4QL − 2QF. MRF(QL,QF) = 100 − 2QL − QF. MRF(QL,QF) = 100 − QL − 2QF.

MRF(QL,QF) = 100 − 2QL − 4QF

Suppose that the inverse demand for a downstream firm is P = 150 − Q. Its upstream division produces a critical input with costs of CU(Qd) = 5(Qd)2. The downstream firm's cost is Cd(Q) = 10Q. When there is no external market for the downstream firm's critical input, the marginal revenue for the downstream firm is: MRd(Q) = 150 − 2Q. MRd(Q) = 150 − Q. MRd(Q) = 140 − 2Q. MRd(Q) = 140 − Q

MRd(Q) = 150 − 2Q.

Management and a labor union are bargaining over how much of a $50 surplus to give to the union. The $50 is divisible up to one cent. The players have one shot to reach an agreement. Management has the ability to announce what it wants first, and then the labor union can accept or reject the offer. Both players get zero if the total amounts asked for exceed $50. Which of the following is a perfect equilibrium? Management requests $49.99, and the labor union accepts $0.01. Management requests $25, and the labor union accepts $25. Management requests $0, and the labor union accepts $50. None of the answers is correct.

Management requests $49.99, and the labor union accepts $0.01.

Which of the following pricing strategies does NOT usually enhance the profits of firms with market power? Marginal cost pricing Price discrimination Block pricing Commodity bundling

Marginal cost pricing

Which of the following pricing strategies does NOT usually enhance the profits of firms with market power? Price matching Cross-subsidies Two-part pricing Marginal cost pricing

Marginal cost pricing

Which of the following market structures would you expect to yield the greatest product variety? Monopoly Monopolistic competition Bertrand oligopoly Perfect competition

Monopolistic competition

_______ occurs when people smoke more after buying life insurance. Adverse selection Moral hazard Asymmetric information Cournot and Bertrand competition

Moral hazard

As firms increase in size, they tend to experience a: decrease in the need for managers. decrease in transaction costs. loss of opportunity cost. None of the answers are correct.

None of the answers are correct.

"An oligopoly is an oligopoly. Firms behave the same no matter what type of oligopoly it is." This statement is true of: Bertrand and Cournot oligopolies. Cournot and Stackelberg oligopolies. Bertrand and Stackelberg oligopolies. None of the answers is correct

None of the answers is correct

Ed just finished an empirical study of oligopoly. He found the following result: "In the examined industry, a firm's demand curve is such that other firms match price increases but do not match price reductions." What kind of oligopoly is the examined industry Sweezy model Cournot model Stackelberg model None of the answers is correct.

None of the answers is correct.

The primary difference between monopolistic competition and perfect competition is: the ease of entry and exit into the industry. the number of firms in the market. Both the ease of entry and exit into the industry and the number of firms in the market are correct. None of the answers is correct

None of the answers is correct.

Which of the following statements is incorrect -As the population rises, the market demand curve shifts to the right -As a greater fraction of the population becomes elderly, the demand for medical services will tend to increase -Changes in the composition of the population affect the demand for a product -None of the statements associated with this question are incorrect

None of the statements associated with this questions are incorrect

A firm has capacity limitations and charges $30 for its service during daily peak times. If the market demand elasticity drops from −3 during peak times to −5 at off-peak times, how much should the firm charge to earn the maximum profit during off-peak times? $20 $21 $24 Not enough information to determine

Not enough information to determine

Oligopoly differs from monopoly as follows: Oligopoly involves a few firms; monopoly involves a single firm. Oligopoly does use advertisement; monopoly does not use advertisement. Oligopoly involves free entry; monopoly involves no free entry. Oligopoly involves a few firms; monopoly involves a single firm and oligopoly involves free entry; monopoly involves no free entry.

Oligopoly involves a few firms; monopoly involves a single firm.

An oligopolist faces a demand curve that is steeper at higher prices than at lower prices. Which of the following is most likely? The firm competes with others in the Cournot fashion. Other firms match price increases but do not match price reductions. Other firms match price reductions but do not match price changes. The firm competes with others in the Bertrand fashion.

Other firms match price increases but do not match price reductions.

When a country opens up trade and becomes a net importer or net exporter of good X, then The domestic consumers of good X lose surplus and the domestic producers of good X gain surplus, but consumers lose more than producers gain. Both consumers and producers gain surplus Both consumers and producers lose surplus Overall surplus is increased

Overall surplus is increased

In the long run, perfectly competitive firms produce a level of output such that: P = MC. P = minimum of AC. P = MC and P = minimum of AC. None of the answers is correct

P = MC and P = minimum of AC

Which of the following is true under monopoly? P > ATC P > MC P = MR P = ATC

P > MC

Which of the following pricing policies does NOT extract the entire consumer surplus from the market? First-degree price discrimination Peak load pricing Two-part pricing Block pricing

Peak load pricing

Suppose you produce wooden desks, and government legislation protecting the spotted owl has made it more expensive for you to purchase wood. What do you expect to happen to the equilibrium price and quantity of wooden desks? -price and quantity will increase -price will increase but quantity will decrease -price and quantity will decrease -price will decrease but quantity will increase

Price will increase but quantity will decrease

Which of the following is NOT a measure of market structure? Entry conditions Four-firm concentration ratio Herfindahl-Hirschman index Pricing behavior

Pricing behavior

Advertising is an aspect of a firm's: performance. structure. environment. conduct

Producer and consumer surpluses are measures of industry performance. market structure. firm conduct. None of the answers are correct.

With a linear inverse demand function and the same constant marginal costs for both firms in a homogeneous product Stackelberg duopoly, which of the following will results Profits of leader > Profits of follower. QL = 2QF. PL > PF. Profits of leader > Profits of follower and QL = 2QF

Profits of leader > Profits of follower and QL = 2QF

The domestic demand and supply for sugar are Qd = 40,000 − 200P and QSD = 10,000 + 300P. The foreign supply is QSF = 20,000 + 100P. What is the total supply of sugar in the domestic market? Q = 50,000 + 100P Q = 30,000 + 400P Q = 15,000 + 200P Q = 10,000 + 300P

Q = 30,000 + 400

Consider a market consisting of two firms where the inverse demand curve is given by P = 500 − 2Q1 − 2Q2. Each firm has a marginal cost of $50. Based on this information, we can conclude that aggregate quantity in the different equilibrium oligopoly models will follow which of the following orderings? QCollusion < QStackelberg < QCournot < QBertrand QCollusion < QCournot < QStackelberg < QBertrand QBertrand < QCollusion < QCournot < QStackelberg QBertrand < QStackelberg < QCournot < QCollusion

QCollusion < QCournot < QStackelberg < QBertrand

Insider trading is prohibited by the: Security Exchange Act. Lanham Act. Clayton Act. Robinson-Patman Act.

Security Exchange Act.

Which of the following is antitrust legislation? Sherman Act Securities and Exchange Act Lanham Act Sherman Act and Lanham Act

Sherman Act

Which of the following institutions may result in hold-up Vertical integration Piece rates Long-term contracts Spot markets

Spot markets

Which would you expect to make the highest profits, other things equal? Bertrand oligopolist Cournot oligopolist Stackelberg leader Stackelberg follower

Stackelberg leader

If consumers expect future prices to be higher -they substitute current purchases for future purchases of perishable products -stockpiling will happen when products are durable in nature -the position of the demand will not change -the demand for automobiles today will not change

Stockpiling will happen when products are durable in nature

Consider an antique auction where bidders have independent private values. There are two bidders, each of whom perceives that valuations are uniformly distributed between $100 and $1,000. One of the bidders is Sue, who knows her own valuation is $200. What is Sue's optimal bidding strategy in a second-price, sealed-bid auction Submit a bid of $150. Submit a bid of $200. Submit a bid that is less than $150. Yell "mine" when the bid reaches $150

Submit a bid of $200.

An oligopolist has a marginal revenue curve that jumps down at 500 units of output. What kind of oligopoly does the firm most likely belong to? Sweezy Cournot Stackelberg Bertrand

Sweezy

Rindistan currently runs a current account deficit with Ackland. to make up the difference, Rindistan prints up more of its curreny, the Rael. Both countries have flexible exchange rates. The increased supply of Raels will cause The Rael to appreciate relative to Ackland's currency The Rael to depreciate relative to Ackland's currency Neither an appreciation nor a depreciation of the Rael Unable to determine

The Rael to depreciate relative to Ackland's currency

In order to eliminate the inefficiency brought about by a monopoly, the government wants to impose a price ceiling on the monopoly. What is the optimal price to be imposed? The competitive price The competitive price, unless it is below ATC. The competitive price, unless it is below MR. The competitive price, unless it is above ATC.

The competitive price, unless it is below ATC

Which of the following is NOT a condition for a firm to engage in price discrimination? Consumers are partitioned into two or more types, with one type having a more elastic demand than the other. The firm has a means of identifying consumer types. The consumers are sincere in revealing their true natures. There is no resale market for the good

The consumers are sincere in revealing their true natures.

The short-run response of quantity demanded to a change in price is usually: -the same as the long-run response -less than the long-run response -greater than the long-run response -greater than the long -run response -none of the statements is correct

The demand for which of the following commodities is likely to be most price inelastic -food -hamburgers -big macs -sandwiches

When the government imposes an effective price ceiling on a monopolist, what will be sure to happen in the short run? The dollar price will increase. The dollar price will fall. There will be a shortage of the product. There will be a surplus of the product.

The dollar price will fall.

When a country opens up trade and becomes a net exporter of good X, then The domestic consumers of good X lose surplus and the domestic producers of good X gain surplus, but consumers lose less than producers gain. The domestic consumers and producers of good X both gain surplus The domestic consumers of good X gain surplus and the domestic producers of good X lose surplus, but consumers gain more than producers lose. The domestic consumers of good X gain surplus and the domestic producers of good X lose surplus, but consumers gain less than producers lose

The domestic consumers of good X lose surplus and the domestic producers of good X gain surplus, but consumers lose less than producers gain.

Which of the following statements is true? The market structure of an industry frequently changes over time. Most horizontal mergers are blocked by the government. Most U.S. industries are perfectly competitive. Most U.S. industries are monopolies.

The market structure of an industry frequently changes over time.

A student figured out that the HHI for an industry was 13,000. What is the proper conclusion? The market is monopolistically competitive. The market is close to perfectly competitive. The market is served by a monopoly. The student made some computational errors

The student made some computational errors

Management and a labor union are bargaining over how much of a $50 surplus to give to the union. The $50 is divisible up to one cent. The players have one shot to reach an agreement. Management has the ability to announce what it wants first, and then the labor union can accept or reject the offer. Both players get zero if the total amounts asked for exceed $50. Which of the following is true? There are multiple Nash equilibria. ($25, $25) is a Nash equilibrium. A Nash equilibrium is also a perfect equilibrium. There are multiple Nash equilibria, and ($25, $25) is a Nash equilibrium.

There are multiple Nash equilibria, and ($25, $25) is a Nash equilibrium.

Which of the following features is common to both perfectly competitive markets and monopolistically competitive markets? Firms produce homogeneous goods. There is free entry. Long-run profits are zero. There is free entry and long-run profits are zero.

There is free entry and long-run profits are zero

Consider the monopoly in the figure below with price regulated at $2 per unit. Monopoly profits at the regulated price (assuming the presence of fixed costs) are: $12. $16. $5 There is insufficient information to determine the monopoly profits.

There is insufficient information to determine the monopoly profits.

Which of the following is an outside incentive that forces managers to put forth maximal effort? Revenue-sharing contracts Performance bonuses Threat of takeovers Flat fees

Threat of takeovers

Which of the following is NOT a transaction cost associated with using inputs? Time spent negotiating labor contracts with union workers Opportunity costs of negotiating the price of renting machines Wages paid to labor Costs of searching for a new supplier of machines

Wages paid to labor

Sue and Jane own two local gas stations. They have identical constant marginal costs, but earn zero economic profits. Sue and Jane constitute: a Sweezy oligopoly. a Cournot oligopoly. a Bertrand oligopoly. None of the answers is correct.

a Bertrand oligopoly.

Suppose a new contracting environment that requires clearing fewer legal hurdles is considered. This new contract will result in an increase in the marginal cost and a longer optimal contract. an increase in the marginal cost and a shorter optimal contract. a decrease in the marginal cost and a longer optimal contract. a decrease in the marginal cost and a shorter optimal contract.

a decrease in the marginal cost and a longer optimal contract

The minimum legal price that can be charged in a market is: -a price floor -a price ceiling -non-pecuniary price -full economic price

a price floor

An import quota is: a fixed fee that an importing firm must pay the domestic government in order to have the legal right to sell the product in the domestic market. the fee an importing firm must pay to the domestic government on each unit it brings into the domestic market. a restriction limiting the quantity of imported goods that can legally enter a domestic market. None of the statements are correct.

a restriction limiting the quantity of imported goods that can legally enter a domestic market.

When there are economies of scope between two products which are separately produced by two firms, merging into a single firm can: accomplish an increase in sales. accomplish a reduction in costs. lead to an increase in cost. lead to a reduction in sales

accomplish a reduction in costs.

The long-run average cost curve defines the minimum average cost of producing alternative levels of output, allowing for optimal selection of: fixed factors of production. only one factor of production. all factors of production. sunk cost factors of production.

all factors of production.

Good X is a normal good if an increase in income leads to -an increase in the supply for good X -an increase in the demand for good X -a decrease in the demand for good X -a decrease in the supply for good X

an increase in the demand for good X

The difference between average total costs and average variable costs is: marginal cost. average fixed cost. fixed cost. None of the statements is correct.

average fixed cost.

Which of the following is not a supply shifter -level of technology -prices of inputs -average income level -weather

average income level

The purpose of randomized pricing is to reduce: consumer price information only. competitor price information only. both customer and competitor information about price. the firm's pricing inflexibility.

both customer and competitor information about price.

R&D is an aspect of a firm's: performance. conduct. structure. environment

conduct.

A consumer's reservation price is the price at which a: consumer prefers to search rather than purchasing at the lowest observed price. consumer prefers to purchase at the lowest observed price rather than to engage in another search. consumer is indifferent between searching again and purchasing at the lowest observed price. producer is indifferent between selling the product and not selling the product.

consumer is indifferent between searching again and purchasing at the lowest observed price.

The purpose of the Clean Air Act (1970) was to reduce market power. control negative externalities. improve competition across international markets. help make information easily obtainable for producers and consumers.

control negative externalities.

The elasticity which shows the responsiveness of the demand for a good due to changes in the price of a related good is the -own price elasticity -income elasticity -log-linear elasticity -cross-price elasticity

cross-price elasticity

When marginal cost curve is below an average cost curve, average cost is: increasing with output. declining with output. not varying with output. None of the statements is correct.

declining with output

If demand increases, then the -demand curve shifts to the left -demand curve shifts to the right -equilibrium prices goes down -equilibrium quantity goes down

demand curve shifts to the right

If the own price elasticity of demand is infinite in absolute value, then: -demand is perfectly elastic -the demand curve is vertical -consumers do not respond at all to changes in price -the demand curve is vertical and consumers do not respond at all to changes om price

demand is perfectly elastic

If a manager is not the owner, the manager: receives the full benefit of good decisions. bears the full cost of bad decisions. does not receive the full benefit nor the full cost of his or her decisions. None of the statements is correct.

does not receive the full benefit nor the full cost of his or her decisions

When the government imposes an excise tax on foreign imports: domestic consumers are harmed. domestic firms benefit. domestic firms are harmed. domestic consumers are harmed and domestic firms benefit

domestic consumers are harmed and domestic firms benefit.

Point B in the figure below is: efficient since it produces 20 units of output at the lowest possible cost. efficient since it produces 10 units of output at the lowest possible cost. inefficient since it produces 20 units of output at a cost greater than the minimum cost. inefficient since it produces 10 units of output at a cost greater than the minimum cost.

efficient since it produces 10 units of output at the lowest possible cost.

As we move up along a linear demand curve, the price elasticity of demand becomes more: elastic. inelastic. log-linear. variable

elastic

If the demand function for a particular good is Q = 50 − 4P, then demand at a price of $10 is: elastic. unit elastic. inelastic. Elasticity cannot be determined.

elastic

Economics

exists because of scarcity

Consider the following information for a simultaneous move game: If you advertise and your rival advertises, you each will earn $5 million in profits. If neither of you advertises, you will each earn $10 million in profits. However, if one of you advertises and the other does not, the firm that advertises will earn $15 million and the non-advertising firm will earn $1 million. If you and your rival plan to be in business for 10 years, then the Nash equilibrium is: for each firm to advertise every year. for neither firm to advertise in early years, but to advertise in later years. for each firm to not advertise in any year. for each firm to advertise in early years, but not advertise in later years

for each firm to advertise every yea

Consider the following information for a simultaneous move game: If you advertise and your rival advertises, you each will earn $5 million in profits. If neither of you advertises, you will each earn $10 million in profits. However, if one of you advertises and the other does not, the firm that advertises will earn $15 million and the non-advertising firm will earn $1 million. If you and your rival plan to hand your business down to your children (and this "bequest" goes on forever), then a Nash equilibrium when the interest rate is zero is: for each firm to not advertise until the rival does, and then to advertise forever. for your firm to never advertise. for your firm to always advertise when your rival does. for each firm to advertise until the rival does not advertise, and then not advertise forever.

for each firm to not advertise until the rival does, and then to advertise forever.

Suppose the demand for good X is given by Qdx = 10 + axPx + ayPy is positive then: -goods y and x are complements -goods y and x are inferior goods -goods y and x are normal goods -good y and x are substitutes

goods y and x are substitutes

A lump-sum tariff imposed on foreign competitors will: always remove foreign competitors from the market. increase the profits of domestic firms when demand is high. have no impact on domestic firms' profits when demand for domestic goods is high. decrease the profits of domestic firms when demand is high.

have no impact on domestic firms' profits when demand for domestic goods is high.

A lump-sum tariff imposed on foreign competitors will: always remove foreign competitors from the market. increase the profits of domestic firms when demand is high. have no impact on domestic firms' profits when demand for domestic goods is high. decrease the profits of domestic firms when demand is high.

have no impact on domestic firms' profits when demand for domestic goods is high.

Monopolistic competition is characterized by: heterogeneous products. employing labor from a perfectly competitive labor market. no free entry. large markets.

heterogeneous products.

Conglomerate integration reduces the transaction costs of acquiring inputs. improves cash flow by exploiting the cyclical nature of different product lines. exploits economies of scope by merging the production of similar products. All of the statements are correct

improves cash flow by exploiting the cyclical nature of different product lines.

An excise tariff imposed on foreign competitors will: increase domestic firms' profits at all levels of demand. increase domestic firms' profits only when demand is high. increase domestic firms' profits only when demand is low. have no impact on domestic firms' profits when demand for domestic goods is high

increase domestic firms' profits at all levels of demand.

Demand is perfectly elastic when the absolute value of the own price elasticity of demand is: -zero -one -infinite -unknown

infinite

Advertising provides consumers with information about the underlying existence or quality of a product. These types of advertising messages are called -persuasive advertising -informative advertising -green advertising - influential advertising

informative advertising

The special demand structure that induces a firm to use a cross-subsidization strategy is: perfect substitution among products. imperfect substitution among products. independent demand for products. interdependent demand for products

interdependent demand for products

A firm will maximize the present value of future profits by maximizing current profits when the:

interest rate is larger than the growth rate in profits and both are constant

Rent seeking: results in less market share for the rent seekers. involves lobbyists influencing government policies to benefit their interests. results in more negative externalities. None of the statements are correct.

involves lobbyists influencing government policies to benefit their interests.

The own price elasticity of demand for apples is -1.5. If the price of apples falls by 6 percent, what will happen to the quantity of apples demanded -it will increase by 4 percent -it will increase 9 percent -it will fall 4 percent -it will fall 6 percent

it will increase 9 percent

Which of the following pairs of goods are probably complements -electricity and natural gas ' -butter and margarine -steak and chicken -ketchup and French fries

ketchup and French fries

The average product of labor depends on how many units of labor are used. capital are used. labor and capital are used. None of the statements is correct

labor and capital are used.

If A and B are complements, an increase in the price of good A would:

lead to a decrease in demand for B

If A and B are substitute goods, an increase in the price of good A would: -have no effect on the quantity demanded of B -lead to an increase in demand for B -lead to a decrease in demand for B -none of the statements associated with this question are correct

lead to an increase in demand for B

If the marginal product per dollar spent on capital is less than the marginal product per dollar spent on labor, then in order to minimize costs the firm should use: less capital and more labor. less labor and more capital. less labor and less capital. more labor and more capital

less capital and more labor.

A linear demand function exhibits: constant demand elasticity. more elastic demand as output increases. less elastic demand as output increases. insufficient information to determine.

less elastic demand as output increases

The inverse demand in a Cournot duopoly is P = a − b(Q1 + Q2), and costs are C1 (Q1 ) = c1Q1 and C2(Q2) = c2Q2. The government has imposed a per-unit tax of $t on each unit sold by each firm. The tax revenue is t times the total output of the two firms should there be no sales tax. less than t times the total output of the two firms should there be no sales tax. greater than t times the total output of the two firms should there be no sales tax. None of the answers is correct

less than t times the total output of the two firms should there be no sales tax.

The optimal bid in a first-price, sealed-bid auction with independent private values is to bid: the true value of the item. more than the true value of the item. less than the true value of the item. the true value of the item and more than the true value of the item, depending upon whether value estimates are affiliated.

less than the true value of the item.

The causal view of an industry is that: market structure causes firms to behave in a certain way. market performance causes firms to have a certain structure. market performance causes firms to behave in a certain way. behavior causes firms to have a certain structure.

market structure causes firms to behave in a certain way.

Differentiated goods are a feature of a: perfectly competitive market. monopolistically competitive market. monopolistic market. monopolistically competitive market and monopolistic market.

monopolistically competitive market

A coordination problem usually occurs in situations where there is: no Nash equilibrium in a game. a unique, but undesirable Nash equilibrium. a unique, secure strategy for both players. more than one Nash equilibrium.

more than one Nash equilibrium.

The isoquants are normally drawn with a convex shape because inputs are: not perfectly substitutable. perfectly substitutable. perfect complements. normal goods.

not perfectly substitutable

Since most consumers spend very little on salt, a small increase in the price of salt will Reduce quantity demanded by a large amount Reduce quantity demanded by a small amount Increase quantity demanded by a large amount Increase quantity demanded by a small amount

not reduce quantity demanded by very much

Spot markets are an INEFFICIENT way for the firm to purchase inputs if: opportunism is a problem. suppliers engage in hold-up. profit sharing is used to compensate managers. opportunism is a problem and suppliers engage in hold-up.

opportunism is a problem and suppliers engage in hold-up.

A Lerner index of 0 suggests: monopoly. monopolistic competition. oligopoly. perfect competition.

perfect competition

There is a market supply curve in a: perfectly competitive market. monopolistically competitive market. monopolistic market. perfectly competitive market and monopolistically competitive market.

perfectly competitive market.

New firms have incentive to enter an industry when there is (are)

positive economic profits

The creation of a new product is referred to as: process innovation. independent research and development. product innovation. patent disclosure.

product innovation

To ensure quality, piece-rate plans must usually be accompanied by: quality control mechanisms. time clocks. spot checks. profit-sharing plans.

quality control mechanisms.

Consider a Cournot duopoly with the following inverse demand function: P = 50 − 0.2Q1 − 0.2Q2. The firms' marginal costs are identical and are given by MCi (Qi ) = 2. Based on this information, firm 1 and 2's reaction functions are: r1 (Q2) = 120 − 0.5Q1 and r2(Q1 ) = 120 − 0.5Q2. r1 (Q2) = 120 − 0.5Q2 and r2(Q1 ) = 120 − 0.5Q1 . Q1 = 240 − 0.75Q2 and Q2 = 240 − 0.75Q1 . Q1 = 240 − 0.5Q2 and Q2 = 240 − 0.5Q1

r1 (Q2) = 120 − 0.5Q2 and r2(Q1 ) = 120 − 0.5Q1

Consider a Stackelberg duopoly with the following inverse demand function: P = 100 − 2Q1 − 2Q2. The firms' marginal costs are identical and are given by MCi (Qi ) = 2. Based on this information, the follower's reaction function is: rF(QL) = 24.5 − 0.5QF. QL = 49 − 0.5QF. rF(QL) = 24.5 − 0.5QL. QF = 49 − 0.25QL.

rF(QL) = 24.5 − 0.5QL.

Generally, revenue-based incentive schemes: reduce incentives to produce low-quality products. increase incentives to minimize costs. reduce worker productivity. reduce incentives to produce low-quality products and increase incentives to minimize costs.

reduce incentives to produce low-quality products.

The main purpose of antitrust policy is to: reduce market power. control negative externalities. help make information easily obtainable for producers and consumers. All of the statements associated with this question are correct.

reduce market power

A consumer spends more time searching for a good when her reservation price is increased. reduced. fixed. None of the statements is correct

reduced.

Consider a market characterized by a Herfindahl-Hirschman index of 5,000. One of the firms in this market has a Lerner index of 0.89 and is considering a horizontal merger with a competing firm. Based on this information, it is likely that the U.S. Department of Justice will: approve the merger since the industry is not concentrated and the firm proposing to merge has little market power. reject the merger since the industry is highly concentrated, even though the firm proposing the merger has little market power. reject the merger since the industry is highly concentrated and the firm proposing the merger has significant market power. None of the answers are correct.

reject the merger since the industry is highly concentrated and the firm proposing the merger has significant market power.

... two prostitutes came to the king (Solomon) and stood before him. One of them said, 'My lord, this woman and I live in the same house. I had a baby while she was there with me. The third day after my child was born, this woman also had a baby. We were alone; there was no one in the house but the two of us. During the night this woman's son died because she lay on him. So she got up in the middle of the night and took my son from my side while I your servant was asleep. She put him by her breast and put her dead son by my breast. The next morning, I got up to nurse my son—and he was dead! But when I looked at him closely in the morning light, I knew that it wasn't the son I had borne.' The other woman said, 'No! The living one is my son; the dead one is yours.' The king said, 'Bring me a sword.' So they brought a sword for the king. He then gave an order: 'Cut the living child in two and give half to one and half to the other.' The woman whose son was alive was filled with compassion for her son and said to the king, 'Please, my lord, give her the living baby. Do not kill him; she is his mother.' But the other said, 'Neither I nor you shall have him. Cut him in two!' Then the king gave his ruling: 'Give the living baby to the first woman. Do not kill him; she is his mother.'" 1 Kings 3:16-27 (NIV). King Solomon's actions in this passage illustrate the concept of signaling. screening. moral hazard. adverse selection

screening

Suppose you are a risk-neutral manager attempting to hire a new sales manager. All of the workers in the market have the same ability to manage and sell, but they differ with respect to the wage at which they are willing to work for your company. The market for sales managers is composed of three types of individuals: 85 percent are willing to work for $75,000 and 15 percent are willing to work for $85,000. The first interviewee is only willing to work for $85,000. If the human resource director spends five hours interviewing each candidate and the opportunity cost of this director's time is $500, then the director should: search again since the expected benefit of an additional search exceeds the cost. stop searching since the expected benefit of an additional search is less than the cost. search again since the expected benefit of an additional search is less than the cost. stop searching since the expected benefit of an additional search exceeds the cos

search again since the expected benefit of an additional search exceeds the cost.

Suppose a consumer has determined that her reservation price, R, is $75. The expected benefit of an additional search at this reservation price is $25. Based on this information we can conclude that: search costs are $25 per search. this consumer will reject any price above $25. this consumer will accept any price below $75. search costs are $25 per search, and this consumer will accept any price below $75.

search costs are $25 per search, and this consumer will accept any price below $75.

For a wood furniture manufacturer, an increase in the cost of lumber will cause the supply curve to: Become flatter Become steeper Shift to the left Shift to the right

shift to the left

For a steel factory, a decrease in the cost of electricity to the plant will cause the supply curve to: -become flatter -shift to the left -shift to the right -become parallel to the price axis

shift to the right

In a competitive market, the market demand is Qd = 60-6P and the market supply is Qs = 4P. A price ceiling of $3 will result in a -shortage of 30 units -shortage of 15 units -surplus of 30 units -surplus of 12 units

shortage of 30 units Qd=60-6(3)= 42; Qs=4(3) =12 42-12=30

Consider the monopoly in the figure below with price regulated at $2 per unit. The regulated price will result in a: surplus of 2 units. shortage of 2 units. surplus of 5 units. shortage of 5 units

shortage of 5 units

A relationship-specific exchange occurs when a partnership is dissolved. specialized investments are important. a partnership is initiated. shareholders receive dividends

specialized investments are important.

The presence of minimal specialized investments relative to contracting costs suggests that the optimal input procurement method is: spot exchange. vertical integration. contract. vertical integration or contract

spot exchange

As additional consumers obtain the benefits of a pure public good, such as national defense, the benefits to the existing consumers will: decrease. increase. stay the same. increase in the short run, but decrease in the long run.

stay the same.

If the cross-price elasticity between goods X and Y is positive, we know the goods are: inferior goods complements inelastic. substitutes.

substitutes

Costs that are forever lost after they have been paid are: production costs. fixed costs. sunk costs. variable costs.

sunk costs

In order to minimize the cost of producing a given level of output, a firm manager should use more inputs when: that input's price rises. that input's price falls. that input's price remains the same. the prices of other inputs fall.

that input's price falls.

The principal's goals are NOT in line with the goals of: any other principal. the agents. the firms. the consumers

the agents.

if demand is perfectly inelastic, then: -the own price elasticity of demand is infinite in absolute value -a small increase in price will lead to a situation where none of the good is purchased -the demand curve is vertical -none of the statements is correct

the demand curve is vertical

Other things held constant, the greater the price of a good -the higher the quantity demanded -the greater the consumer surplus -the lower the consumer surplus

the lower the consumer surplus

Suppose a monopolist has positive fixed costs and constant marginal costs. If the government regulates a monopoly's price to marginal cost, in the long run: the monopolist will earn a profit if ATC > MC. the monopolist will exit the industry. the monopolist will earn a profit if ATC > P. the monopolist will earn zero profits.

the monopolist will exit the industry.

Fixed costs exist only in: the long run. capital-intensive markets. the short run. labor-intensive markets.

the short run.

The marginal product of an input is defined as the change in: -average output attributable to the last unit of an input. -total output attributable to the last unit of an input. -total input attributable to the last unit of an output. -average output attributable to the last unit of an output

total output attributable to the last unit of an input.

Accounting profits are:

total revenue minus total cost

To avoid the problem of double marginalization: transfer prices must be set that maximize the overall value of the firm rather than the profits of the upstream division. firms should put more emphasis on vertical integration. firms should engage in two-part pricing. firms should engage in commodity bundling, unless it is possible to engage in either first-or second-degree price discrimination.

transfer prices must be set that maximize the overall value of the firm rather than the profits of the upstream division.

Revenue sharing tries to induce worker effort by linking: worker compensation to profits. worker compensation to revenues. worker output to profits. worker output to revenues

worker compensation to revenues.

A potential problem with piece-rate plans is that: workers will produce a large quantity. workers have no incentive to work hard. it is difficult for managers to control. workers may stress quantity instead of quality.

workers may stress quantity instead of quality

If you advertise and your rival advertises, you each will earn $3 million in profits. If neither of you advertises, you will each earn $7 million in profits. However, if one of you advertises and the other does not, the firm that advertises will earn $10 million and the non-advertising firm will earn $1 million. If you and your rival plan to be in business for 15 years, then the Nash equilibrium is for: you and your rival to not advertise in any year. you and your rival to advertise every year. neither firm to advertise in early years, but to advertise in later years. each firm to advertise in early years, but not advertise in later years.

you and your rival to advertise every year

The exchange rate between the Japanese Yen and the UK Pound is 12.70 ¥/£. It costs $1.72 US to buy a pound. How many Yen does it take to purchase $1 US? ¥21.85 ¥0.05 ¥7.38 ¥0.14

¥7.38

Which of the following profit functions exhibits a Leontief production function? π = P × K0.75L0.50 − 20L − 35K π = P × min(2L, 5K) − 20L − 35K π = P × (3K + 4L) − 20L − 35K π = P × (3K0.5 + 4L0.5) 1/0.2 − 20L − 35K

π = P × min(2L, 5K) − 20L − 35K

Consider a market consisting of two firms where the inverse demand curve is given by P = 500 − 2Q1 − 2Q2. Each firm has a marginal cost of $50. Based on this information, we can conclude that aggregate profits in the different equilibrium oligopoly models will follow which of the following orderings? πBertand > πCollusion > πStackelberg > πCournot πCollusion > πCournot > πStackelberg > πBertand πCollusion > πStackelberg > πCournot > πBertand None of the answers is correct.

πCollusion > πCournot > πStackelberg > πBertand

If a monopolist claims his profit-maximizing markup factor is 3, what is the corresponding price elasticity of demand? −1.5. −2.0. −2.5. −3.0.

−1.5.

In a monopoly where the marginal revenue and price are, respectively, given by $0.50 and $2, the price elasticity of demand is: −0.75. −1. −5/4. −4/3.

−4/3.

Compute the marginal revenue when the price elasticity of demand is −0.10. −9P, meaning marginal revenue is negative and 9 times greater than price. 9P, meaning marginal revenue is positive and 9 times greater than price. −3P, meaning marginal revenue is negative and 3 times greater than price. 3P, meaning marginal revenue is positive and 3 times greater than price

−9P, meaning marginal revenue is negative and 9 times greater than price.


Conjuntos de estudio relacionados

Adult Nursing 2 - Chapter 20 PrepU

View Set

Unit 2 Quiz MISY-3310-W01 - MANAGEMENT INFORMATION SYSTEMS

View Set

Dendritic Cells and Antigen Processing (9)

View Set

Chapters 1 General Insurance Terms

View Set

Fern gametophytes are characterized by all of the following EXCEPT

View Set

International Business SW1/Chapter 1 HS16

View Set